Simplify. -(-6w + x - 3y)

Answers

Answer 1

Answer: 6w - x + 3y

Step-by-step explanation:


Related Questions

Dylan’s boat can carry 40 people across a river. Last month, 2504 people road on Dylan’s boat. What is the least number of trips that Dylan could have made across that river.

Answers

The required least number of trips that Dylan would have made across the river is 62.

Given that,
Dylan’s boat can carry 40 people across a river. Last month, 2504 people rode on Dylan’s boat. What is the least number of trips that Dylan could have made across that river is to be determined.

What is arithmetic?

In mathematics, it deals with numbers of operations according to the statements. There are four major arithmetic operators, addition, subtraction, multiplication, and division,

Here,
Total people road  = 2504
Maximum people can road at single trip = 40
Least number of trip = 2504 / 40
Least number of trip = 62

Thus, the required least number of trips that Dylan would have made across the river is 62.

Learn more about arithmetic here:

https://brainly.com/question/11559160

#SPJ1

Martin and Isabelle go bowling. Each game costs $10, and they split that cost. Martin has his own bowling shoes, but Isabelle pays $3 to rent shoes.Which graph shows a proportional relationship? Explain why.

Answers

We have the following:

Martin's graph is good and correct, although it is not totally straight, but the relationship that it keeps is totally proportional.

On the other hand, Isabelle's graph, although it is totally straight, is wrong, because she must start from 3, which is the rental value of the shoes, and her graph starts at 0, therefore it is wrong, despite of which shows a proportional relationship.

Therefore the correct answer is Martin's graph.

Answer:

Step-by-step explanation:

help meeeeeeeeee pleaseee !!!!!

Answers

The solution to the composite function is as follows;

(f + g)(x) = x² + 3x + 5(f - g)(x) = x² - 3x + 5(f. g)(x) = 3x³ + 15x(f / g)(x) = x² + 5 / 3x

How to solve composite function?

The composition of a function is an operation where two functions say f and g generate a new function say h in such a way that h(x) = g(f(x)).

If we are given two functions, it is possible to create or generate a “new” function by composing one into the other.

Composite functions are when the output of one function is used as the input of another.

In other words, a composite function is generally a function that is written inside another function.

Therefore,

f(x) = x² + 5

g(x) = 3x

Hence, the composite function can be solved as follows:

(f + g)(x) = f(x) + g(x)  = x² + 5 + 3x = x² + 3x + 5

(f - g)(x) = f(x) - g(x)  = x² + 5 - 3x = x² - 3x + 5

(f. g)(x) = f(x) . g(x) = (x² + 5)(3x) = 3x³ + 15x

(f / g)(x) = f(x) / g(x) = x² + 5 / 3x

learn more composite function here: brainly.com/question/24464747

#SPJ1

Cylinder A has radius r, height h, and a volume of 10 pi cubic units. Cylinder B hastwice the radius and twice the height.hATBWhat is the volume of cylinder B?I2r2h

Answers

Volume of a cylinder:

[tex]V=h*r^2*\pi[/tex]

For cylinder A:

[tex]10\pi cm^3=h*r^2*\pi[/tex]

For cylinder B:

[tex]V_B=2h*(2r)^2*\pi[/tex]

Simplify the equation for volumen of cylinder B:

[tex]\begin{gathered} V_B=2h*4r^2*\pi \\ V_B=8*(h*r^2*\pi) \end{gathered}[/tex]

in the equation for the volume of cylinder A you have the value of h*r^2*π:

[tex]\begin{gathered} V_B=8*(10\pi cm^3) \\ V_B=80\pi cm^3 \end{gathered}[/tex]Then, the volume of cylinder B is 80π cubic centimeters.

Solve 6 < x + 5 < 11

Answers

we have the following:

[tex]\begin{gathered} 6

2. Consider drawing a card at random from a standard deck of cards,Part A: Determine the probability that the card is a spade, given that it is black,Part B: Determine the probability that the card is red, given that it is a heart,Part C: Determine the probability that the card is an ace, given that it is black.Part D: Determine the probability that the card is a queen given that it is a face card,

Answers

Consider drawing a card at random from a standard deck of cards,

Part A: Determine the probability that the card is a spade, given that it is black,

Part B: Determine the probability that the card is red, given that it is a heart,

Part C: Determine the probability that the card is an ace, given that it is black.

Part D: Determine the probability that the card is a queen given that it is a face card,

we have 52 cards

A standard 52-card deck comprises 13 ranks in each of the four French suits: clubs (♣), diamonds (♦), hearts (♥) and spades (♠)

so

Part A: Determine the probability that the card is a spade, given that it is black,

If the card is black, that means the possible outcomes are 26 cards

so

P=13/26

P=0.5

Part B: Determine the probability that the card is red, given that it is a heart,

if the card is a heart, that means, the possible outcomes are 13

so

P=13/13

P=1

because all the cards that are heart are red

Part C: Determine the probability that the card is an ace, given that it is black.

if the card is black the possible outcomes are 26

therefore

P=2/26

P=1/13

Part D: Determine the probability that the card is a queen given that it is a face card

A projectile is fired vertically upwards and can be modeled by the function h(t)= -16t to the second power+600t +225 during what time interval will the project I’ll be more than 4000 feet above the ground round your answer to the nearest hundredth

Answers

Given:

[tex]h(t)=-16t^2+600t+225[/tex]

To find the time interval when the height is about more than 4000 feet:

Let us substitute,

[tex]\begin{gathered} h(t)\ge4000 \\ -16t^2+600t+225\ge4000 \\ -16t^2+600t+225-4000\ge0 \\ -16t^2+600t-3775\ge0 \end{gathered}[/tex]

Using the quadratic formula,

Here, a= -16, b=600, and c= -3775

[tex]\begin{gathered} t=\frac{-b\pm\sqrt[]{b^2-4ac}}{2a} \\ =\frac{-600\pm\sqrt[]{600^2-4(-16)(-3775)}}{2(-16)} \\ =\frac{-600\pm\sqrt[]{360000^{}-241600}}{-32} \\ =\frac{-600\pm\sqrt[]{118400}}{-32} \\ =\frac{-600\pm40\sqrt[]{74}}{-32} \\ =\frac{-75\pm5\sqrt[]{74}}{-4} \\ t=\frac{-75+5\sqrt[]{74}}{-4},x=\frac{-75-5\sqrt[]{74}}{-4} \\ t=7.99709,t=29.5029 \end{gathered}[/tex]

So, the interval is,

[tex]8.00\le\: t\le\: 29.50[/tex]

What is the slope of this horizontal line from 10-13 minutes?

Answers

We are asked to determien the slope of the line between 10 and 13 minutes. Since this is a horizontal line, it's slope is 0.

1: 9 11. The cost for a group of people to go to the movies is given by the expression 9a + 5b, where a is the number of adults and b is the number of children. What are the variables of this expression? of of A. 9 and 5 B. a and b C. 9a and 5b D. + and x

Answers

the variables are

a and b

where

a -----> is the number of adults

b-------> is the number of children.

answer is option B

If TW =6, WV =2, and UV =25, find XV to the nearest hundredth.

Answers

TW = 6

WV = 2

UV = 25

XV = ?

XV/UV = WV/TV

XV/25 = 2 /(6 + 2)

XV = 2(25)/7

XV = 50/7

XV = 7.1428

Rounded to the nearest hundredth

XV = 7.14

Calculate the slope (2,-5) and (4,3)

Answers

Answer:

Slope = 4

Step-by-step explanation:

The slope of a line can be calculated using the following formula:

[tex] \frac{y2 - y1}{x2 - x1} [/tex]

From the question can put the points as:

(2, -5) as (x1, y1)

and

(4, 3) as (x2, y2)

Therefore, we can put in the values into the formula to solve for the slope.

[tex] \frac{3 - ( - 5)}{4 - 2} \\ = \frac{3 + 5}{2} \\ = \frac{8}{2} \\ = 4[/tex]

Given points are [tex](x_1,y_1) = (2,-5) and (x_2,y_2) = (4,3)[/tex]

Now , using the formula of slope,m = [tex] y_2-y_1/x_2-x_1[/tex]

=> 3-(-5)/4-2

=> 8/2

=> 4

Hence the required slope of given two ordered pairs is 4

I NEED CORRECT ANSWER 100 POINTS ONLY ANSWER CORRECTLY

A line passes through the points (7,9) and (10,1). What is its equation in point-slope form?

Use one of the specified points in your equation. Write your answer using integers, proper fractions, and improper fractions. Simplify all fractions.

Answers

Answer:

[tex]y-9=-\dfrac{8}{3}(x-7)[/tex]

Step-by-step explanation:

[tex]\boxed{\begin{minipage}{4.4cm}\underline{Slope Formula}\\\\Slope $(m)=\dfrac{y_2-y_1}{x_2-x_1}$\\\\where $(x_1,y_1)$ and $(x_2,y_2)$ \\are two points on the line.\\\end{minipage}}[/tex]

Define the given points:

(x₁, y₁) = (7, 9)(x₂, y₂) = (10, 1)

Substitute the defined points into the slope formula:

[tex]\implies \textsf{slope}\:(m)=\dfrac{1-9}{10-7}=-\dfrac{8}{3}[/tex]

[tex]\boxed{\begin{minipage}{5.8 cm}\underline{Point-slope form of a linear equation}\\\\$y-y_1=m(x-x_1)$\\\\where:\\ \phantom{ww}$\bullet$ $m$ is the slope. \\ \phantom{ww}$\bullet$ $(x_1,y_1)$ is a point on the line.\\\end{minipage}}[/tex]

Substitute the found slope and one of the points into the point-slope formula:

[tex]\implies y-9=-\dfrac{8}{3}(x-7)[/tex]

Please help me I don’t know how to do this

Answers

We have a point (4,-9) it moves to (9,-14)

(4+x = 9, -9+y= -14)

x = 9-4

x = 5

y = -14 +9

y = -5

We are moving to the right 5 and down 5

We want to move the point (-9,-8) exactly the same way

(-9+5, -8-5)

(-4, -13)

(-4, -13)

what are the three terms and 4x - 2y + 3

Answers

Solution

We have the following expression:

[tex]4x-2y+3[/tex]

Here we have 3 terms:

[tex]4x,\text{ -2y and 3}[/tex]

Variable terms:

[tex]4x,-2y[/tex]

Constant term

[tex]3[/tex]

in exponential growth functions the base of the exponent must be greater than 1.how would the function change if the base of the exponent were1? how would the function change if the base of the exponents were between 0 and 1

Answers

It’s now 10 to p 1 =11

15 = a/3 - 2
what is a? ​

Answers

Answer: a is 51

Step-by-step explanation:

Hope this help.

Answer:

a==51

Step-by-step explanation:

15=a/3-2

a/3-2+2=15+2

a/3=17

a=17*3

a=51

The volume, V, of a cube with edge length s cm is given by the equation V=s3.Is the volume of a cube with edge length s=3 greater or less than the volume of a sphere with radius 3?If a sphere has the same volume as a cube with edge length 5, estimate the radius of the sphere?Compare the outputs of the two volume functions when the inputs are 2?

Answers

We have that the volume of sphere is

[tex]\begin{gathered} V_s=\frac{4}{3}\pi\cdot r^3 \\ \end{gathered}[/tex]

and the volume of a cube is

[tex]V_c=s^3[/tex]

so if s=r=3. The volume of the sphere is greater.

If they have the same volume, we get that

[tex]\begin{gathered} \frac{4}{3}\pi\cdot r^3=125\rightarrow \\ r^3=\frac{3}{4\cdot\pi}\cdot125\approx29.84\approx30 \\ r=\sqrt[3]{30}\approx3.10 \end{gathered}[/tex]

when s=r=2 we have that

[tex]\begin{gathered} V_s=\frac{4}{3}\pi\cdot8=\frac{32}{3}\pi \\ V_c=8 \end{gathered}[/tex]

so the volume of the sphere is greater

Find fractional notation of 87.5%

Answers

To find the fractional notation, we have to transform 87.5% into a fraction. To do that, we just have to divide the percentage by 100.

[tex]\frac{87.5}{100}[/tex]

Then, we multiply each part by 10.

[tex]\frac{87.5\times10}{100\times10}=\frac{875}{1000}[/tex]

At last, we simplify the fraction by 125 to get 7/8.

[tex]\begin{gathered} \frac{875}{125}=7 \\ \frac{1000}{125}=8 \end{gathered}[/tex]Hence, the given percent, in fractional notation, is 7/8.

Which equation represents a line which is perpendicular to the line y=-5/4x-4?A. 4y−5x=−4B. 5x+4y=−8C. 4x−5y=15D.4x+5y=40

Answers

The slope of a line, m, comes in the equation as the coefficient of x.

In the given equation, m= -5/4. Two perpendicular lines have slopes that are the negative reciprocals of each other.

So, the slope of the perpendicular line will be +4/5.

Between the given options, letter c will be:

4x-5y=15

-5y=15-4x (divided by -5)

y=4/5x-3

Letter C

What is the measure of the angle at the bottom of home plate?

Answers

We will ave the following:

*First: We will determine the sum of all internal angles of the polygon:

[tex](n-2)\cdot180\Rightarrow(5-2)\cdot180=3\cdot180[/tex][tex]=540[/tex]

*Second: Now, that we know that the sum of all internal angles will be 540°, the following is true:

[tex]90+90+135+135+\alpha=540[/tex]

Now, we solve for alpha [The angle]:

[tex]\Rightarrow\alpha=540-135-135-90-90\Rightarrow\alpha=90[/tex]

So, the measure of the angle at the bottom is 90°.

10. A $152,000 home has an assessment rate of 52% and a tax rateof $48 per $1,000. Use the effective tax method to calculate theproperty tax .Hint: When you determine the effective tax rate, round the rateto three places.

Answers

Given

$152,000

52% assessment rate

$48 per $1,000

Procedure

First, let's calculate the assessment rate.

[tex]152000\cdot0.52=79040.0[/tex]

Now let's calculate the taxes

[tex]79040.0\cdot\frac{48}{1000}=3793.92[/tex]

Property taxes are equal to $3,793.92.

O GRAPHS AND FUNCTIONSWriting an equation for a function after a vertical and horizo

Answers

Given:

The point (0,0) lies on the graph f(x) and (4,-3) lies on the graph h(x).

To find:

We need to find the equation for the function h(x).

Explanation:

Consider the translation point which is translated horizontally a unit and vertically as b units.

[tex](x^{\prime},y^{\prime})\rightarrow(x+a,y+b)[/tex]

The point (4,-3) can be written as follows.

[tex](4,-3)\rightarrow(0+4,0-3)[/tex]

We get the function h(x) after f(x) translated horizontally 4 units right and vertically 3 units down.

The function can be written as follows.

[tex]h(x)=f(x-4)-3[/tex][tex]\text{Replace x=x-4 in f(x)=}\sqrt[]{x\text{ }}\text{ and substitute in the equation.}[/tex]

[tex]h(x)=\sqrt[]{x-4}-3[/tex]

Final answer:

[tex]h(x)=\sqrt[]{x-4}-3[/tex]

Simplify (5x + 7) - (x + 2)

Answers

You have the following expression:

(5x + 7) - (x + 2)

in order to simplify the previous expression, eliminate parenthesis and take into account that if a parenthesis is preceeded by a minus sign, when you elminate th eparenthesis the sign inside change to the opposite, just as follow:

(5x + 7) - (x + 2) =

5x + 7 - x - 2 =

5x - x + 7 - 2 =

4x + 5

Hence, the simplified expression is 4x + 5

A square room has a floor area of 49 square meters. The height of the room is 8 meters. What is the total area of all four walls?​

Answers

The total area of all four walls is 224 square meters.

According to the question,

We have the following information:

A square room has a floor area of 49 square meters.

So, we have:

Area of square = 49 square meters

Side*side = 49

Side = [tex]\sqrt{49}[/tex] m

Side of the square = 7 m

Now, the side of the floor will be the width of the wall.

So, we have the width of the wall = 7 m.

The height of the room is 8 meters.

It means that the height of the wall is 8 m.

Area of 1 rectangular wall = length*width

Area of wall = 8*7

Area of 1 wall = 56 square meters

Now, the are of 4 walls will be (4*56) square meters or 224 square meters.

Hence, the total are of all four walls is 224 square meters.

To know more about total area of all four walls here

https://brainly.com/question/16836796

#SPJ1

Solve for x8x-11=6x-5Simplify your answer as much as possible

Answers

Solve the given equation for x as shown below

[tex]\begin{gathered} 8x-11=6x-5 \\ \Rightarrow8x-11-6x=6x-5-6x \\ \Rightarrow2x-11=-5 \\ \Rightarrow2x-11+11=-5+11 \\ \Rightarrow2x=6 \\ \Rightarrow\frac{2x}{2}=\frac{6}{2} \\ \Rightarrow x=3 \end{gathered}[/tex]Therefore, the solution to 8x-11=6x-5 is x=3.

Determine the value of k for which f(x) is continuous.

Answers

These are the conditions of the continuity in a function:

First, the value of x must have an image.

Second, the lateral limits must be equal:

[tex]\lim_{x\to a^+}f(x)=\lim_{x\to a^-}f(x)[/tex]

Finally, the value of the limit must be equal to the image of x. This means that:

[tex]f(a)=\lim_{x\to a^}f(x)[/tex]

In this case, we must find a value of k that can make the two lateral limits equal in x =3:

[tex]\lim_{x\to3^+}x^2+k=\lim_{x\to3^-}kx+5[/tex]

We can solve these two limits easily by replacing the x with the value of 3

[tex]3^2+k=3k+5[/tex][tex]\begin{gathered} 9+k=3k+5 \\ 4=2k \\ k=2 \end{gathered}[/tex]

Finally, we can see that the answer is k=2.

f(x) = x2 + 1 g(x) = 5 – x

(f + g)(x) =

x to the power of 2 – x + 6

then (f – g)(x) =??

Answers

The function operation ( f - g )( x ) in the functions f(x) = x² + 1 and g(x) = 5 - x is x² + x - 4.

What is the function operation ( f - g )( x ) in the given functions?

A function is simply a relationship that maps one input to one output. Each x-value can only have one y-value.

Given the data in the question;

f(x) = x² + 1g(x) = 5 - x( f - g )( x ) = ?

To find ( f - g )( x ), replace the function designators in ( f - g ) with the actual functions.

( f - g )( x ) = f( x ) - g( x )

( f - g )( x ) = ( x² + 1 ) - ( 5 - x )

Remove the parenthesis using distributive property

( f - g )( x ) = ( x² + 1 ) - ( 5 - x )

( f - g )( x ) = x² + 1 - 5 + x

Collect and add like terms

( f - g )( x ) = x² + x + 1 - 5

( f - g )( x ) = x² + x - 4

Therefore, the function operation ( f - g )( x ) is x² + x - 4.

Learn more about functions here: brainly.com/question/2541698

#SPJ1

The required function would be (f – g)(x) = x² + x - 4.

What is the function?

A mathematical expression that defines the connection between two variables is considered a function.

The given functions following as

f(x) = x² + 1 and g(x) = 5 - x

We have to determine the function (f – g)(x).

(f – g)(x)  = f(x) - g(x)

Substitute the values of functions f(x) = x² + 1 and g(x) = 5 - x in the function (f - g).

(f – g)(x) = (x² + 1) - (5 - x)

Open the parenthesis and apply the arithmetic operation,

(f – g)(x)  = x² + 1 - 5 + x

Rearrange the terms likewise and combine them,

(f – g)(x) = x² + x + 1 - 5

Apply the subtraction operation to get

(f – g)(x) = x² + x - 4

Therefore, the required function would be (f – g)(x) = x² + x - 4.

To learn more about the functions click here:

brainly.com/question/12431044

#SPJ1

For her phone service, Mai pays a monthly fee of $19, and she pays an additional $0.04 per minute of use. The least she has been charged in a month is$70.28. What are the possible numbers of minutes she has used her phone in a month?

Answers

We have a phone service fee which can be divided in:

- A fixed fee of $19 per month.

- A variable fee of $0.04 per minute, so that the cost for m minutes is 0.04*m.

We can add the two fees to express the total cost in function of the minutes as:

[tex]C(m)=19+0.04m[/tex]

For a month where the cost is C(m) = 70.28, we can calculate the minutes as:

[tex]\begin{gathered} C(m)=70.28 \\ 19+0.04m=70.28 \\ 0.04m=70.28-19 \\ 0.04m=51.28 \\ m=\frac{51.28}{0.04} \\ m=1282 \end{gathered}[/tex]

Answer: if she pays at least $70.28, she has talked at least m = 1282 minutes per month.

Instructions: Find the value of the trigonometric ratio. Makesure to simplify the fraction if needed.

Answers

Answer:

sin C = 3/5

Explanation:

Given:

CB = 32

AC = 40

AB = 24

To find:

sin C

To determine sinC, we will apply the sine ratio:

[tex]\begin{gathered} sin\text{ C = }\frac{opposite}{hypotenuse} \\ \\ oppoite\text{ =side opposite the angle = AB = 24} \\ hyp\text{ = 40} \end{gathered}[/tex][tex]\begin{gathered} sin\text{ C}=\text{ }\frac{24}{40} \\ \\ sin\text{ C}=\text{ }\frac{3}{5} \end{gathered}[/tex]

Drew has a video game with five differentchallenges. He sets the timer to play his gamefor 10.75 minutes. He spends the same amountof time playing each challenge. How long doesDrew nlay the fifth challenge?

Answers

For each game, Drew spends 10.75 minutes, this means in total Drew spends

[tex]5\cdot10.75\text{ minutes}[/tex]

this product gives

[tex]5\cdot10.75=53.75\text{ minutes}[/tex]

then, in the fifth challenge Drew spends 53.75 minutes

Other Questions
Determine the value of x Round results to an appropriate number of significant digits Answer this fraction based Question I will make you btainliest & provide you 50 points of the following, which is the most influential in deterring international standardization of advertising? multiple choice question. legal forces physical forces sociocultural forces economic forces determine the number of real solutions for the following quadratic equation using the discriminate Two drops of mercury each has a charge on 2.42 nC and a voltage of 293.97 V. If the two drops are merged into one drop, what is the voltage on this drop? a state trooper stopped a defendant for speeding five miles per hour over the speed limit on an interstate highway. initially, the trooper only intended to issue the defendant a warning, but when the defendant refused the trooper's request for a consent search of the car for drugs, the trooper retrieved his narcotics-detection dog from the patrol car and walked around the defendant's car. the dog alerted at the trunk. based on that alert, the trooper searched the trunk and found several grams of cocaine in the defendant's locked suitcase. the defendant was arrested. the entire incident took less than ten minutes, and it was recorded on a video camera located in the patrol car. prior to his jury trial, the defendant's attorney filed a motion to suppress the cocaine based on an illegal search and seizure under the fourth amendment. should the defendant's motion be granted? eric patterson earns $81,000 a year. his monthly expenses total $5,100. what is the minimum amount of money that he should set aside in an emergency fund? what are the methods for structuring and presenting information called?(1 point) responses classification strategies classification strategies organizational strategies organizational strategies narratives and arguments narratives and arguments facts and research how will each of the following changes in demand and/or supply affect equilibrium price and equilibrium quantity in a competitive market? that is, do price and quantity rise, fall, or remain unchanged, or are the answers indeterminate because they depend on the magnitudes of the shifts? Could you help me with this please is from apex Consider the functions below.Represent the interval where both functions are increasing on the number line provided. Select all the correct answers.Imagine that you have been asked to give a graduation speech. Which elements should you avoid when writing the speech?- a strictly formal, argumentative tone- the graduating students' accomplishments- funny quotes from other students- unpleasant remarks about a teacher- simple sentence constructions what is the process of providing a user with permission including access levels and abilities such as file access, hours of access, and amount of allocated storage space? Ashleys Internet service is terribly unreliable in fact on any given day theres a 60% chance that her Internets connection will be lost at some point that day what is the probability that her Internet service is not broken for seven days in a row inner a fraction or round your answer to four decimal places if necessary. if x=10 units, then what is the volume of the cube Select the graph for the solution of the open sentence. Click until the correct graph appears. Ix| + 3 > 3 The Lyon Restaurant Survey provides food, decor, and service ratings for some of the top restaurants across the United States. For 186 restaurants located in Boston, the average price of a dinner, including one drink and tip, was 48.60 Dollars. You are leaving on a business trip to Boston and will eat dinner 23 of these restaurants, randomly selected. Your company will reimburse you for a maximum of 50 dollars per dinner. Business associates familiar with these restaurants have told you that the meal cost at one-third of these restaurants will exceed 50 dollars.a. What is the probability that none of the meals will exceed the cost covered by your company?b. What is the probability that at most 12 of the meals will exceed the cost covered by your company? What is the probability that between 4 and 8 of the meals will exceed the cost covered by your company? c. Calculate the expected number of restaurants that will exceed the cost covered by your company. d. Calculate the probability of the first question by using the binomial distribution approximation. Therefore, in this case we will consider the possibility of repetition in the randomly selected restaurants. Define p=r/N as the success probability.N is the size of the population. r is the number of elements considered as successes in the population. e. Calculate the probability of the second question by using the binomial disribution approximation. f. Calculate the probability of the third question by using the binomial disribution approximation. g. Calculate the expected number of the fourth question by using the binomial disribution aproximation A washer and a dryer cost $765 combined. The washer costs $85 less than the dryer. What is the cost of the dryer? What is troubling about the circumstance when a coil is stationary and a magnet moves as compres to when a magnet is stationary and the coil moves? Find the pairs (LOTS OF POINTS!)